Home Practice
For learners and parents For teachers and schools
Textbooks
Full catalogue
Leaderboards
Learners Leaderboard Classes/Grades Leaderboard Schools Leaderboard
Pricing Support
Help centre Contact us
Log in

We think you are located in United States. Is this correct?

5.3 Remainder theorem

5.3 Remainder theorem (EMCGV)

Remainder theorem

Given the following functions:

  • \(f(x) = x^{3} + 3x^{2} + 4x + 12\)
  • \(k(x) = x - 1\)
  • \(g(x) = 4x^{3} -2x^{2} + x -7\)
  • \(h(x) = x + 2\)
  1. Determine \(\dfrac{f(x)}{k(x)}\) and \(\dfrac{g(x)}{h(x)}\).
  2. Write your answers in the general form: \(a(x)=b(x).Q(x) + R(x)\).
  3. Determine \(f(1)\) and \(g(-2)\).
  4. What do you notice?
  5. Consider the degree of the quotient and the remainder - is there a rule?
  6. What conclusions can you draw?
  7. Write a mathematical equation to describe your conclusions.
  8. Complete the following sentence: a cubic function divided by a linear polynomial gives a quotient with a degree of \(\ldots \ldots\) and a remainder with a degree of \(\ldots \ldots\), which is called a constant.

The Remainder theorem

A polynomial \(p(x)\) divided by \(cx - d\) gives a remainder of \(p\left(\dfrac{d}{c}\right)\).

In words: the value of the remainder \(R\) is obtained by substituting \(x = \frac{d}{c}\) into the polynomial \(p(x)\).

\[R = p \left( \dfrac{d}{c} \right)\]

NOTE: PROOF NOT FOR EXAMS

Let the quotient be \(Q(x)\) and let the remainder be \(R\). Therefore we can write:

\begin{align*} p(x) &= (cx - d) \cdot Q(x) + R \\ \therefore p \left( \dfrac{d}{c} \right) &= \left[c\left( \dfrac{d}{c} \right) - d\right] \cdot Q \left( \dfrac{d}{c} \right) + R \\ &= (d - d)\cdot Q \left( \dfrac{d}{c} \right) + R \\ &= 0 \cdot Q \left( \dfrac{d}{c} \right) + R \\ &= R\\ \therefore p \left( \dfrac{d}{c} \right) &= R \end{align*}

Worked example 7: Finding the remainder

Use the remainder theorem to determine the remainder when \(p(x) = 3x^{3} + 5x^{2} - x + 1\) is divided by the following linear polynomials:

  1. \(x + 2\)
  2. \(2x -1\)
  3. \(x + m\)

Determine the remainder for each linear divisor

The remainder theorem states that any polynomial \(p(x)\) that is divided by \(cx - d\) gives a remainder of \(p\left(\dfrac{d}{c}\right)\):

  1. \begin{align*} p(x) &= 3x^{3} + 5x^{2} - x + 1 \\ p(-2) &= 3 \left( -2 \right)^{3} + 5\left( -2 \right)^{2} - \left( -2 \right) + 1 \\ &= 3 \left( -8 \right) + 5\left( 4 \right) +2 + 1 \\ &= -24 + 20 + 3 \\ \therefore R &= -1 \end{align*}
  2. \begin{align*} p(x) &= 3x^{3} + 5x^{2} - x + 1 \\ p \left( \frac{1}{2} \right) &= 3\left( \frac{1}{2} \right)^{3} + 5\left( \frac{1}{2} \right)^{2} - \left( \frac{1}{2} \right)+ 1 \\ &= 3\left( \frac{1}{8} \right) + 5\left( \frac{1}{4}\right) - \left( \frac{1}{2}\right) + 1 \\ &= \frac{3}{8} + \frac{5}{4} + \frac{1}{2} \\ &= \frac{3}{8} + \frac{10}{8} + \frac{4}{8} \\ \therefore R &= \frac{17}{8} \end{align*}
  3. \begin{align*} p(x) &= 3x^{3} + 5x^{2} - x + 1 \\ p(m) &= 3\left( -m \right)^{3} + 5\left( -m \right)^{2} - \left( -m \right) + 1 \\ \therefore R &= -3m^{3} + 5m^{2} + m + 1 \end{align*}

Worked example 8: Using the remainder to solve for an unknown variable

Given that \(f(x) = 2x^{3} + x^{2} + kx + 5\) divided by \(2x - 3\) gives a remainder of \(9\frac{1}{2}\), use the remainder theorem to determine the value of \(k\).

Use the remainder theorem to determine the unknown variable \(k\)

From the remainder theorem we know that \(f\left(\frac{3}{2}\right) = 9\frac{1}{2}\) and we can therefore solve for \(k\):

\begin{align*} f(x) &= 2x^{3} + x^{2} + kx + 5 \\ f\left( \frac{3}{2} \right) &= 2 \left( \frac{3}{2} \right)^{3} + \left( \frac{3}{2} \right)^{2} + k\left( \frac{3}{2} \right) + 5 \\ 9\frac{1}{2} &= 2 \left( \frac{27}{8} \right) + \left( \frac{9}{4} \right) + k\left( \frac{3}{2} \right) + 5 \\ 9\frac{1}{2} &= \frac{27}{4} + \frac{9}{4} + \frac{3k}{2} + 5 \\ 9\frac{1}{2} &= \frac{36}{4} + \frac{3k}{2} + 5\\ \therefore 9\frac{1}{2} - 9 - 5 &= \frac{3k}{2} \\ - 4\frac{1}{2} &= \frac{3k}{2} \\ - \frac{9}{2} \times \frac{2}{3} &= k \\ \therefore -3 &= k \end{align*}

Write the final answer

Therefore \(k = -3\) and \(f(x) = 2x^{3} + x^{2} - 3x + 5\).

Remainder theorem

Textbook Exercise 5.4

Use the remainder theorem to determine the remainder \(R\) when \(g(x)\) is divided by \(h(x)\):

\begin{align*} g(x) &= x^{3} + 4x^{2} + 11x - 5 \\ h(x) &= x - 1 \end{align*}

\begin{align*} g(x) &= x^{3} + 4x^{2} + 11x - 5 \\ g(1) &= (1)^{3} + 4(1)^{2} + 11(1) - 5 \\ &= 1 + 4 + 11 - 5 \\ \therefore R &= 11 \end{align*}

\begin{align*} g(x) &= 2x^{3} - 5x^{2} + 8 \\ h(x) &= 2x - 1 \end{align*}

\begin{align*} g(x) &= 2x^{3} - 5x^{2} + 8 \\ g \left( \frac{1}{2} \right) &= 2\left( \frac{1}{2} \right)^{3} - 5\left( \frac{1}{2} \right)^{2} + 8 \\ &= 2\left( \frac{1}{8} \right) - 5\left( \frac{1}{4} \right) + 8 \\ &= \frac{1}{4} - \frac{5}{4} + 8 \\ \therefore R &= 7 \end{align*}

\begin{align*} g(x) &= 4x^{3} + 5x^{2} + 6x - 1 \\ h(x) &= x + 2 \end{align*}

\begin{align*} g(x) &= 4x^{3} + 5x^{2} + 6x - 1 \\ g (-2) &= 4(-2)^{3} + 5(-2)^{2} + 6(-2) - 1 \\ &= -32 +20 -12 - 1 \\ \therefore R &= -25 \end{align*}

\begin{align*} g(x) &= -5x^{3} - x^{2} -10x + 9 \\ h(x) &= 5x + 1 \end{align*}

\begin{align*} g(x) &= -5x^{3} - x^{2} -10x + 9 \\ g \left( -\frac{1}{5} \right) &= -5\left( -\frac{1}{5} \right)^{3} - \left( -\frac{1}{5} \right)^{2} -10\left( -\frac{1}{5} \right) + 9 \\ &= -5\left( -\frac{1}{125} \right) - \left( \frac{1}{25} \right) + 2 + 9 \\ &= \frac{1}{25} - \frac{1}{25} + 11 \\ \therefore R &= 11 \end{align*}

\begin{align*} g(x) &= x^{4} + 5x^{2} + 2x - 8 \\ h(x) &= x + 1 \end{align*}

\begin{align*} g(x) &= x^{4} + 5x^{2} + 2x - 8 \\ g (-1) &= (-1)^{4} + 5(-1)^{2} + 2(-1) - 8 \\ &= 1 + 5 - 2 - 8 \\ &= 6 -10 \\ \therefore R &= -4 \end{align*}

\begin{align*} g(x) &= 3x^{5} - 8x^{4} + x^{2} + 2 \\ h(x) &= 2 - x \end{align*}

\begin{align*} h(x) &= 2 - x \\ \text{If } x = 2: \quad h (2) &= 2 - 2 = 0 \\ g(x) &= h(x) \cdot Q(x) + R \\ g(2) &= h(2) \cdot Q(2) + R \\ \therefore g(2) &= 0 \cdot Q(2) + R \\ g(2) &= 0 + R \\ \therefore R &= 3(2)^{5} - 8(2)^{4} + (2)^{2} + 2 \\ &= 3(32) - 8(16) + 6 \\ &= 96 - 128 + 6 \\ \therefore R &= -26 \end{align*}

\begin{align*} g(x) &= 2x^{100} - x - 1 \\ h(x) &= x + 1 \end{align*}

\begin{align*} g(x) &= 2x^{100} - x - 1\\ g (-1) &= 2(-1)^{100} - (-1) - 1 \\ &= 2 + 1 - 1 \\ \therefore R &= 2 \end{align*}

Determine the value of \(t\) if \(x^{3} + tx^{2} + 8x + 21\) divided by \(x + 1\) gives a remainder of \(\text{16}\).

\begin{align*} a(x) &= x^{3} + tx^{2} + 8x + 21 \\ b(x) &= x + 1 \\ R &= 16 \\ a (-1) &= (-1)^{3} + t(-1)^{2} + 8(-1) + 21 \\ \therefore 16 &= -1 +t -8 + 21 \\ \therefore t &= 4 \end{align*}

Calculate the value of \(m\) if \(2x^{3} - 7x^{2} + mx - 26\) is divided by \(x - 2\) and gives a remainder of \(-\text{24}\).

\begin{align*} a(x) &= 2x^{3} - 7x^{2} + mx - 26 \\ b(x) &= x - 2 \\ R &= -24 \\ a (2) &= 2(2)^{3} - 7(2)^{2} + m(2) - 26 \\ \therefore -24 &= 16 -28 +2m -26 \\ 14 &= 2m \\ \therefore m &= 7 \end{align*}

If \(x^{5} - 2x^{3} - kx - 1\) is divided by \(x - 1\) and the remainder is \(-\frac{1}{2}\), find the value of \(k\) .

\begin{align*} a(x) &= x^{5} - 2x^{3} - kx - 1 \\ b(x) &= x - 1 \\ R &= -\frac{1}{2}\\ a (1) &= (1)^{5} - 2(1)^{3} - k(1) - 1 \\ \therefore -\frac{1}{2} &= 1 - 2 - k - 1 \\ \frac{3}{2} &= -k \\ \therefore k &= -\frac{3}{2} \end{align*}

Determine the value of \(p\) if \(18x^{3} + px^{2} - 8x + 9\) is divided by \(2x - 1\) and gives a remainder of \(\text{6}\).

\begin{align*} a(x) &= 18x^{3} + px^{2} - 8x + 9 \\ b(x) &= 2x - 1 \\ R &= 6 \\ a \left( \frac{1}{2} \right) &= 18\left( \frac{1}{2} \right)^{3} + p\left( \frac{1}{2} \right)^{2} - 8\left( \frac{1}{2} \right) + 9 \\ \therefore 6 &= 18 \left( \frac{1}{8} \right) + p \left( \frac{1}{4} \right) -4 + 9 \\ 6 &= \frac{18}{8} + \frac{p}{4} + 5 \\ 1 &= \frac{18}{8} + \frac{p}{4} \\ 4 &= 9 + p \\ \therefore p &= -5 \end{align*}

If \(x^{3} + x^{2} - x + b\) is divided by \(x - 2\) and the remainder is \(2\frac{1}{2}\), calculate the value of \(b\).

\begin{align*} \text{Let } f(x) &= x^{3} + x^{2} - x + b \\ R &= \frac{5}{2} \\ f (2) &= (2)^{3} + (2)^{2} - (2) + b \\ \therefore \frac{5}{2} &= 8 + 4 - 2 + b \\ \frac{5}{2} -10 &= b \\ \therefore b &= - \frac{15}{2} \end{align*}

Calculate the value of \(h\) if \(3x^{5} + hx^{4} + 10x^{2} - 21x + 12\) is divided by \(x - 2\) and gives a remainder of \(\text{10}\).

\begin{align*} a(x) &= 3x^{5} + hx^{4} + 10x^{2} - 21x + 12 \\ b(x) &= x -2 \\ R &= 10 \\ a (2) &= 3(2)^{5} + h(2)^{4} + 10(2)^{2} - 21(2) + 12 \\ \therefore 10 &= 3(32) + 16h + 40 -42 +12 \\ 10 &= 96 + 16h + 10 \\ -96 &= 16h \\ \therefore h &= -6 \end{align*}

If \(x^{3} + 8x^{2} + mx - 5\) is divided by \(x + 1\) and the remainder is \(n\), express \(m\) in terms of \(n\).

\begin{align*} a(x) &= x^{3} + 8x^{2} + mx - 5 \\ b(x) &= x + 1 \\ R &= n \\ a (-1) &= (-1)^{3} + 8(-1)^{2} + m(-1) - 5 \\ \therefore n &= -1 + 8 -m - 5 \\ n &= 2 - m \\ \therefore m &= 2 - n \end{align*}

When the polynomial \(2x^{3} + px^{2} + qx + 1\) is divided by \(x + 1\) or \(x - 4\), the remainder is \(\text{5}\). Determine the values of \(p\) and \(q\).

\begin{align*} a(x) &= 2x^{3} + px^{2} + qx + 1 \\ a(-1) &= 2(-1)^{3} + p(-1)^{2} + q(-1) + 1 \\ \therefore 5 &= -2 + p - q + 1 \\ 6 &= p - q \\ \therefore q &= p - 6 \ldots \ldots (1)\\ a(4) &= 2(4)^{3} + p(4)^{2} + q(4) + 1 \\ \therefore 5 &= 128 + 16p + 4q + 1 \\ -124 &= 16p + 4q \\ \therefore -31 &= 4p + q \ldots \ldots (2)\\ \text{Substitute eqn } (1) & \text{ into eqn } (2) \\ \therefore -31 &= 4p + (p - 6) \\ -25 &= 5p \\ \therefore -5 &= p \\ \text{And } q &= -5 -6 \\ & = -11 \end{align*}